A problem like Fermat's Last Theorem

  • Thread starter kmarinas86
  • Start date
  • Tags
    Theorem
In summary: Fermat's Last Theorem with a^n+b^n=c^n.In summary, there are infinitely many integer solutions to Fermat's Last Theorem with a^n+b^n=c^n, as long as c is between 1 and 2.
  • #1
kmarinas86
979
1
A "problem" like Fermat's Last "Theorem"

Given the equation:

[itex]a^n+b^n=c^n[/itex]
Where:
a, b, and c are integers.
And n is real number greater than 2.

Is there a solution?

edit: a, b, and c are supposed to be non-zero like in Fermat's Last Theorem
 
Last edited:
Physics news on Phys.org
  • #2
Yes. Try picking a, b, and c before you worry about n.
 
  • #3
Yes. Try picking a, b, and c before you worry about n.

But a, b, and c are supposed to be non-zero integers. That's what Fermat's Last Theorem is about.
 
  • #4
Okay, pick them to be nonzero integers. Then worry about n.
 
  • #5
You asked whether there were any positive integers a, b, c, such that, for some n, an+ bjn= cn. Hurkyl's suggestion was that you choose suitable a, b, c, then look for a value of n.
 
  • #6
HallsofIvy said:
You asked whether there were any positive integers a, b, c, such that, for some n, an+ bjn= cn. Hurkyl's suggestion was that you choose suitable a, b, c, then look for a value of n.

That doesn't answer my question though.
 
  • #7
As your first task, let a and b be the same integer.
 
  • #8
That doesn't answer my question though.

Woe is you. These people sure are mean when they don't tell you the solution straight away.
 
  • #9
Fermat's Last Theorem goes like this:

Where a, b, c, and n are all integers:
And where n is an integer greater than 2,

There can be no a, b, c, n, where:
[itex]a^n+b^n=c^n[/itex]

http://www.google.com/search?q=fermat's+last+theorem

What non integer n should I use then? There are literally an infinite amount. I would choose n first because it was already proven that the formula does not work where n is an integer.

Do you understand that it took hundreds of years to prove Fermat's Last Theorem? It looks so simple, but it's not. By experience it is known by mathematicians that there is no such thing as a simple proof "thus far" of Fermat's Last Theorem. Therefore, the answers I have gotten so far have been inadequate.

I haven't found any substantation whatsoever whether or not the no solution answer (which was proven for the n=integer case) applies for the n=real number case.
 
Last edited:
  • #10
Of course it doesn't apply. And I think those people who posted here to help you are more than familiar with the history of FLT, and quite a few might even know about Wiles's proof.

Fix a<b<c and you're asking if a^n+b^n-c^n= 0. Try and find some simple conditions to ensure that there is some n where this happens. Hint: intermediate value theorem, and try to be polite to other people who are trying to help you to help yourself, it really is quite straight forward to find infinitely many a,b,c and n real and greater than 2, as a really big hint try a=b=1 and *any* c larger than 2 for an idea (if you want n>2 then try a little harder, though why n=1.5 won't do I'mi not sure).

NB fixing n can't help you at all, and you are after all trying to show that there is some n for which there are integer solutions, and as you observe there are infinitely many to choose from, so don't choose it first.
 
Last edited:
  • #11
1^2.5 < 1^2.5+1^2.5 < 2^2.5
1^3.5 < 1^3.5+1^3.5 < 2^3.5

In general
1^x < 1^x+1^x < 2^x
for any x greater than 1
where
1 < 2 < 2^x

There is no integer between 1 and 2.

Where x>2:
1^x+1^x < 2^x
1^x+1^x > 1^x

For any:
a^n+b^n=c^n
Where a, b, and c are integers, and n is a real number.
the nth root of (a^n+b^n) must equal an integer.
 
Last edited:
  • #12
Sorry, what does that show?

It is easy to find infinitely many (strictly positive) integer a,b,c and real n>2 with a^n+b^n=c^n using the intermediate value theorem with, so try doing it
 
  • #13
matt grime said:
Sorry, what does that show?

It means:

kmarinas86 said:
1^2.5 < 1^2.5+1^2.5 < 2^2.5
1^3.5 < 1^3.5+1^3.5 < 2^3.5

In general
1^x < 1^x+1^x < 2^x
for any x greater than 1
where
1 < 2 < 2^x

There is no integer between 1 and 2.

Where x>2:
1^x+1^x < 2^x
1^x+1^x > 1^x

This proves that for the equation:
1^x+1^x = c^x, where x>1
c cannot be an integer. It must be a number between 1 and 2. Of course, this is not a formal proof, but a formal proof can be derived from this. In fact, c must be the xth root of 2.
because
1^x+1^x = 2

Also
Any root of 2 cannot ever be less than 1.
And 1^x = 1.
And 2 to any xth root cannot ever be an integer. (for any x > 1)


matt grime said:
It is easy to find infinitely many (strictly positive) integer a,b,c and real n>2 with a^n+b^n=c^n using the intermediate value theorem with, so try doing it

You're right:

If you look at the graph below, you'll see that for x>2 as x approaches infinity, the graph line becomes asymptotic. Soon I will post a graph that has a bunch of lines, maybe dozens, that will probably show the same thing. But I have found, that indeed, there are solutions.
 

Attachments

  • continuousfunction.jpg
    continuousfunction.jpg
    32.2 KB · Views: 397
Last edited:
  • #14
It is very easy to do (to find infinitely many a,b,c integers and n a real larger than 2). We are not lying to you, nor are we pretending not to know the answer when we don't. Do you know what the intermediate value theorem states? That is all that is required, and a little thought (the case a=b=1 surely tells you something, how about making a=b=4?), and infinitely many solutions can be shown to exist by anyone who's done a first course in analysis.Instead of assuming that we're all wrong, why don't you possibly consider that we might be right and try following the hints you've been given? You'll learn more by doing so. As I say, infinitely many examples can be shown to exist by anyone with a first course in analysis behind them.

Let me help even more by pointing out that n=log(1/2)/log(4/5) which is greater than two will have integer solutions to a^n+b^n=c^n.

Indeed, now I come to think of it anyone with high school maths can create arbitrarily many examples. A first course in analysis will explain 'why' this is possible. As I say, it is merely the intermediate value theorem.
 
  • #15
Solution for 3^x+3^x=4^x:

ln(2)/ln(4/3)=x

How to get it:
3^x+3^x=4^x
2*3^x=4^x
2=(4/3)^x
ln(2)=ln(4/3)*x
ln(2)/ln(4/3)=x

a, b, c are all integers, and x is a real number.

Mystery solved :smile:
 

Attachments

  • continuousfunction2.jpg
    continuousfunction2.jpg
    38 KB · Views: 421
  • #16
matt grime said:
It is very easy to do (to find infinitely many a,b,c integers and n a real larger than 2). We are not lying to you, nor are we pretending not to know the answer when we don't. Do you know what the intermediate value theorem states? That is all that is required, and a little thought (the case a=b=1 surely tells you something, how about making a=b=4?), and infinitely many solutions can be shown to exist by anyone who's done a first course in analysis.


Instead of assuming that we're all wrong, why don't you possibly consider that we might be right and try following the hints you've been given? You'll learn more by doing so. As I say, infinitely many examples can be shown to exist by anyone with a first course in analysis behind them.

Let me help even more by pointing out that n=log(1/2)/log(4/5) which is greater than two will have integer solutions to a^n+b^n=c^n.

Indeed, now I come to think of it anyone with high school maths can create arbitrarily many examples. A first course in analysis will explain 'why' this is possible. As I say, it is merely the intermediate value theorem.

It's ok, I figured out on my own before I saw this post :smile:
 

What is Fermat's Last Theorem?

Fermat's Last Theorem is a mathematical problem proposed by Pierre de Fermat in the 17th century. It states that no three positive integers a, b, and c can satisfy the equation an + bn = cn for any integer value of n greater than 2.

Why is Fermat's Last Theorem considered important?

Fermat's Last Theorem is considered important because it remained unsolved for over 350 years, leading to numerous failed attempts and false proofs by some of the most prominent mathematicians in history. Its eventual solution in 1994 by Andrew Wiles was seen as a major breakthrough in the field of mathematics.

What is the significance of Fermat's Last Theorem?

Fermat's Last Theorem has significant implications in number theory and algebraic geometry. Its proof required the development of new mathematical techniques and tools, leading to advancements in these fields. It also showcases the power and beauty of mathematics in solving complex problems.

How was Fermat's Last Theorem finally solved?

The proof of Fermat's Last Theorem was completed by Andrew Wiles in 1994, building upon the work of several mathematicians over the centuries. Wiles used advanced mathematical concepts such as elliptic curves and modular forms to prove the theorem, which required over seven years of dedicated work.

Are there any real-life applications of Fermat's Last Theorem?

Fermat's Last Theorem does not have any direct real-life applications. However, its solution has advanced the field of mathematics and has inspired further research in number theory and algebraic geometry. It also serves as a reminder of the power and potential of human intellect in solving even the most elusive problems.

Similar threads

Replies
15
Views
1K
Replies
1
Views
765
  • Linear and Abstract Algebra
Replies
2
Views
1K
  • Linear and Abstract Algebra
Replies
3
Views
2K
Replies
12
Views
2K
  • Linear and Abstract Algebra
Replies
2
Views
994
  • Linear and Abstract Algebra
Replies
1
Views
2K
  • Linear and Abstract Algebra
Replies
1
Views
1K
Replies
2
Views
1K
  • Linear and Abstract Algebra
Replies
2
Views
907
Back
Top